\[ \boxed{ \left( {x}^{2}+1 \right) {x}^{3}{\frac {{\rm d}^{3}}{{\rm d}{x}^{3}}}y \left( x \right) - \left( 4\,{x}^{2}+2 \right) {x}^{2}{\frac {{\rm d}^{2}}{{\rm d}{x}^{2}}}y \left( x \right) + \left( 10\,{x}^{2}+4 \right) x{\frac {\rm d}{{\rm d}x}}y \left( x \right) -4\, \left( 3\,{x}^{2}+1 \right) y \left( x \right) =0} \]